Find the sum of all integral values of 3 3 3 x − 3 2 x + 1 + 3 x + 1 − 1 9 5 2 where x is an integer.
This section requires Javascript.
You are seeing this because something didn't load right. We suggest you, (a) try
refreshing the page, (b) enabling javascript if it is disabled on your browser and,
finally, (c)
loading the
non-javascript version of this page
. We're sorry about the hassle.
That was INCREDIBLE! Loved this solution and really improved my math skills xD
This solution is perfect. Double Thumbs up.
See, I think waiting for someone else to post a solution before I do is the better strategy. Me posting a solution immediately deters others from doing so.
Solved the same way!!Nice!
Very nice!
Must say, an extremely astute question and an equally wonderful solution!!!!!!!
Here's a semi-open question: Why was it the number 1 9 5 2 at the beginning? What, if anything, was special about 1 9 5 2 here? What conditions did that number, say k , need to satisfy in order for there to be an integer solution using the method above?
Log in to reply
It's one more than a prime number, I guess. But great solution! :D
Log in to reply
Thanks :) (Also, the part at the end, with the quadratic, should work out to an integer root.)
Another good solution would be a bounding argument. for big enough n , we must have that n 3 − ( n − 1 ) 3 > 1 9 5 1 , where n in this case is 3 x − 1 . We find that if x > 3 , then n 3 − ( n − 1 ) 3 > 1 9 5 1 so the expression can never be a perfect cube. Thus we just need to check the values of x = 0 , 1 , 2 , 3 .
in case 2 it should be a=n+1951 and not a=n+1.
Log in to reply
Oops, sorry. That happens when you copy-paste between cases :P
i did same
Tanks Dood for the Awesome sol :)
For x>3 we have ( 3 x − 1 ) 3 − ( 3 x − 2 ) 3 > 1 9 5 1 .That leaves x=3,2,1.Done :D
Could hardly fpllow.Plz xpln!
Log in to reply
The thing under the radical is equal to ( 3 x − 1 ) 3 − 1 9 5 1 .We want this to be an integer, call it a.Then ( 3 x − 1 ) 3 − a 3 = 1 9 5 1
But for x > 3 we would have
( 3 x − 1 ) 3 − ( 3 x − 2 ) 3 > 1 9 5 1 , so we would obviously have no solutions.
Now we are left with checking for 1,2 and 3.
I think the reason this problem would have been tough is if you were using a crappy calculator. Because I noticed for almost all x 's, the formula produced a number that was like, insanely close to a whole number, like at x = 1 2 , the result was 5 3 1 4 3 9 . 9 9 9 9 9 9 9 9 7 6 9 7 3 5 2 0 9 3 9 8 7 5 3 7 1 2 9 3 7 3 2 2 5 9 7 6 0 4 3 2 2 7 4 2 0 1 3 . But I just kept plugging in values. The only real "trick" I used was realizing that x had to be greater than 2 because otherwise the part under the radical would be negative. Plugging in x = 3 produces 2 5 which I guess is the only truly integral solution. I'll be waiting for a good solution though, @Daniel Liu . Nice problem! :D
It's super hot down here at SD...so I don't feel like writing a full solution.
Here are some hints:
Notice the ( a − 1 ) 3 factorization.
1951 is prime
It's fine if the part under the radical (the radicand) is negative, since we're taking a cube root...
Your calculator is giving you a number a tiny bit smaller than 3 x − 1 for large enough values of x . E.g. 3 1 2 − 1 = 5 3 1 4 4 0 . This is because the expression simplifies to 3 ( 3 x − 1 ) 3 − 1 9 5 1 , and for large enough values of x , subtracting 1 9 5 1 doesn't change the cube root very much.
This gives some insight into the actual solution of the problem.
Daniel? I'm waiting for a freaking boss solution... :D
Log in to reply
From now on, I'm waiting for someone else to maybe post a solution before I do. Give others a chance.
Log in to reply
Aw nartz. But you do have one, right? Is it really cool?
Log in to reply
@Finn Hulse – Basically the same as Michael's.
Log in to reply
@Daniel Liu – Oh epic. How did you think this up? Did you just choose a number 1 greater than a prime?
Log in to reply
@Finn Hulse – No, I had to also choose the number such that it actually gives an integer solution. Not all numbers that are one more than a prime work.
Log in to reply
@Daniel Liu – 'Tis a problem. But still, excellent problem. BTW what did you get on USAJMO? Did you make USAMO? :O
Log in to reply
@Finn Hulse – I didn't make USAMO because I took the USAJMO. And I got a 9 on the JMO.
Log in to reply
@Daniel Liu – Wait 9 out of what?
Log in to reply
@Finn Hulse – 9 out of 42.
Log in to reply
@Daniel Liu – Ooh that stings. @Akshaj Kadaveru got a 21/42. :O
Done with my solution :)
Log in to reply
@Michael Tang – EPIC! This solution was so amazing. :D
I did it the same way, and then found that x = 3 works, and then I figured the problem couldn't go on forever, so I stopped at 25.
Log in to reply
Yah. This problem was supposed to be Level 5. @Daniel Liu what are the stats as of right now for who has solved it?
Log in to reply
29 views, 6 attempts, 6 solvers. It's too easy to guess the right solution, I guess.
Log in to reply
@Daniel Liu – Yea this is a problem. @Calvin Lin how many of the 7 solvers currently, do you suppose, actually solved the question instead of trying the first few x and then guessing that there aren't any more solutions? Is there any way to prevent guessing?
Log in to reply
@Daniel Liu – It is a shortcoming of asking for numeral answers (which are easy to evaluate in scale) as opposed to proofs (which are hard to individually judge).
There is no easy way to prevent guessing, or to prevent "I threw it into Python".
@Daniel Liu – @Daniel Liu I did this the same way as @Michael Tang . Is it still 1 9 attempts and 1 9 solvers?
it's really interesting............. :-p
Problem Loading...
Note Loading...
Set Loading...
Suppose that 3 3 x − 3 2 x + 1 + 3 x + 1 − 1 9 5 2 = n 3 for some integer n . Then we can write
3 3 x − 3 ⋅ 3 2 x + 3 ⋅ 3 x − 1 9 5 2 = n 3 .
Now, the critical step is to notice this factorization:
( 3 x − 1 ) 3 − 1 9 5 1 = n 3 .
(Notice the 1 , − 3 , 3 coefficients on the left-hand side, which was my primary motivation.) Then, rearranging gives ( 3 x − 1 ) 3 − n 3 = 1 9 5 1 . Let a = 3 x − 1 , so we have a 3 − n 3 = 1 9 5 1 or
( a − n ) ( a 2 + a n + n 2 ) = 1 9 5 1 .
Normally, we would now proceed to try a bunch of factor pairs of 1 9 5 1 ... but we hope, hope, hope that Daniel was nice to us when writing this problem and won't make us try too many. Indeed, 1 9 5 1 is prime, the easiest of all! :P
Also, note the following lemma:
Lemma. For all real numbers a , n (whether negative or positive), the following inequality holds: a 2 + a n + n 2 ≥ 0 .
Proof. If a , n have the same sign (or one/both of them is zero), then a n ≥ 0 , so
a 2 + a n + n 2 ≥ a 2 + n 2 ≥ 0 + 0 = 0 .
Otherwise, if a , n have the opposite sign, then a n ≤ 0 ; but now we can write
a 2 + a n + n 2 = ( a + n ) 2 − a n ≥ ( a + n ) 2 ≥ 0 .
Thus, in all cases, a 2 + a n + n 2 ≥ 0 .
So, by the Lemma, there are only two cases to try:
(1) Suppose that a − n = 1 and a 2 + a n + n 2 = 1 9 5 1 . We substitute a = n + 1 into the second equation to get ( n + 1 ) 2 + n ( n + 1 ) + n 2 = 1 9 5 1 , which simplifies to
3 ( n 2 + n − 6 5 0 ) = 0 .
This has roots n = 2 5 or n = − 2 6 . But if n = − 2 6 , then a = − 2 5 and the equation 3 x − 1 = − 2 5 has no real solutions. Thus, n = 2 5 , so a = 2 6 , so 3 x − 1 = 2 6 ⟹ x = 3 . The solution x = 3 corresponds to the value 2 5 , since we defined n to be the value of the expression.
(2) Suppose that a − n = 1 9 5 1 and a 2 + a n + n 2 = 1 . We substitute a = n + 1 into the second equation to get ( n + 1 9 5 1 ) 2 + n ( n + 1 9 5 1 ) + n 2 = 1 , which simplifies to
3 n 2 + 5 8 5 3 n + 3 8 0 6 4 0 0 = 0 .
This is a mess, and has no real roots for n , so there are no solutions in this case.
So, the only value of x is x = 3 , which corresponds to the value 2 5 . □